平狄克《微观经济学》(第7版)习题详解(第8章 利润最大化和竞

更新时间:2024-04-16 23:29:01 阅读量: 综合文库 文档下载

说明:文章内容仅供预览,部分内容可能不全。下载后的文档,内容与下面显示的完全一致。下载之前请确认下面内容是否您想要的,是否完整无缺。

Born to win

经济学考研交流群 <<<点击加入

平狄克《微观经济学》(第7版) 第8章 利润最大化和竞争性供给

课后复习题详解

跨考网独家整理最全经济学考研真题,经济学考研课后习题解析资料库,您可以在这里查阅历年经济学考研真题,经济学考研课后习题,经济学考研参考书等内容,更有跨考考研历年辅导的经济学学哥学姐的经济学考研经验,从前辈中获得的经验对初学者来说是宝贵的财富,这或许能帮你少走弯路,躲开一些陷阱。

以下内容为跨考网独家整理,如您还需更多考研资料,可选择经济学一对一在线咨询进行咨询。

1.为什么一个发生亏损的厂商选择继续进行生产而不是关闭? 答:一个发生亏损的厂商选择继续进行生产而不是关闭的原因在于:此时的价格仍然大于平均可变成本,但小于平均成本。具体来讲:

(1)厂商发生亏损是指总收益TR小于总成本TC,此时,如果总收益TR仍然大于可变成本VC,厂商继续生产将会弥补VC,并且可以弥补一部分的固定成本FC,弥补量是TR?VC,从而使损失最小化,因此在短期内,厂商不会关闭而是继续生产。

(2)停止营业点是指一个已经投入生产的企业,在生产中总有这样一点,当根据利润最大化原则确定的产量大于这一点所代表的产量时,仍可以继续生产,小于这一点所代表的产量时,就只好关闭。

一个已经投入生产的企业是否必须关闭的条件不在于它是否盈利,而在于它关闭后的亏损与生产时的亏损哪种情况更大。如果关闭后的亏损比生产时的亏损更大,则应继续生产;如果生产时的亏损比关闭后的亏损更大,则必须关闭。实际上关闭后也是有亏损的,其亏损就是固定成本。因此,是否关闭就视生产时的亏损是否大于固定成本而定,若不大于,就可继续生产,若大于,就必须停止营业。企业的停止营业点可用图8-4说明:图中N点即平均可变成本最低点就是企业停止营业点。

图8-4 停止营业点

(3)当市场决定的价格为P2时,均衡产量为Y2,恰好等于N点所表示的产量。这时,总亏损为面积BFJN,即等于总固定成本。此时,厂商的平均收益AR等于平均可变成本AVC,厂商可以继续生产,也可以不生产,也就是说,厂商生产或不生产的结果都是一样的。这是因为,如果厂商生产的话,则全部收益只能弥补全部的可变成本,不变成本得不到985/211历年真题解析,答案,核心考点讲义,你想要的都在这→ 经济学历年考研真题及详解

Born to win

经济学考研交流群 <<<点击加入

任何弥补。如果厂商不生产的话,厂商虽然不必支付可变成本,但是全部不变成本仍然存在。由于在这一均衡点上,厂商处于关闭企业的临界点,所以,该均衡点也被称作停止营业点或关闭点。

2.解释为什么行业长期供给曲线不是行业长期边际成本曲线。 答:(1)供给曲线是指每一可能的价格下厂商将生产的产量。供给曲线有短期和长期之分。

(2)在完全竞争条件下,单个厂商是价格接受者。在短期,企业生产规模固定不变,厂商通过选择产量水平以实现利润最大化。厂商将增加产量直到价格等于边际成本,但如果价格低于平均可变成本,它则会不生产。因此一个厂商的短期供给曲线是短期边际成本曲线(位于最小平均可变成本点之上的部分)。

(3)但在长期,厂商无固定成本,成本全部是可变的。由于厂商能够调整生产规模,进入或退出某一个行业,因此厂商可以把生产规模调整到使短期边际成本与市场价格相等的水平上。随着长期价格的变动,厂商逐渐变动自己的要素组合来最小化其生产成本。所以,长期供给曲线是厂商不断从一组短期边际成本曲线向另一组短期边际成本曲线调整的结果,而不是长期边际成本曲线(位于最小平均总成本的点之上的部分)。此外,在长期,每个厂商利润都为零,因此任何位于MC?AC的产出水平都是不可行的。

3.在长期均衡时,行业中所有厂商的经济利润为零。为什么?

答:在完全竞争条件下,行业没有任何的进入和退出障碍。因此,在实现长期均衡的动态过程中,如果行业有正的经济利润,就会有厂商进入。新厂商的进入使得行业供给曲线向右移动,均衡价格下降,从而使得行业中厂商的经济利润下降。而如果大量厂商的进入使经济利润为负,必然会促使一些厂商退出该行业。因此,最终的平衡状态必然是行业中所有厂商的经济利润为零,从而没有厂商进入或者退出行业。

部分厂商比其他厂商的会计利润更高,是因为他们能够获得比较便宜的资源,如果考虑机会成本,即它并不自己生产而是把这些资源以更高价格卖给其他厂商的收益,那么厂商的经济利润同样也是为零的。因此,只要厂商所在的行业是竞争性的,那么长期中其经济利润一定为零。

4.经济利润和生产者剩余有什么不同?

答:经济利润与生产者剩余之间的差额就是生产的固定成本FC。具体来讲:

(1)经济利润是指属于企业所有者的、超过生产过程中所运用的所有要素的机会成本的一种收益,它等于总收益TR与总成本TC之间的差额,即TR?TC。

(2)生产者剩余指厂商在提供一定数量的某种产品时实际接受的总支付和愿意接受的最小总支付之间的差额。它通常用市场价格线以下,SMC曲线以上的面积来表示,如图8-5中的阴影部分面积所示。其原因在于:在生产中,只要价格大于边际成本,厂商进行生产总是有利的。这时,厂商就可以得到生产者剩余。

985/211历年真题解析,答案,核心考点讲义,你想要的都在这→ 经济学历年考研真题及详解

Born to win

经济学考研交流群 <<<点击加入

图8-5 生产者剩余

(3)在短期内,由于固定成本是无法改变的,所以,总边际成本必然等于总可变成本。因此,生产者剩余也可以用厂商的收益和总可变成本的差额来定义。

总之,生产者剩余可以用厂商的总收益TR和总可变成本TVC的差额来定义,即

,即TR?TVC。而经济利润等于总收益减去总成本(总可变成本加上总固定成本)

TR?TVC?TFC。所以,经济利润与生产者剩余之间的差额就是生产的总固定成本TFC。

5.当厂商知道在长期它们的经济利润为零时,它们为什么还进入这个行业? 答:其原因可以从以下两个方面来分析:

(1)实现长期均衡是一个动态过程,需要很长时间。在长期均衡时,厂商得到的经济利润为零,而在短期内可能有相当的利润。厂商进入行业是无成本的,因此厂商的收益依赖于他所能获得的短期利润。

长期均衡概念反映厂商行为的方向,即在当前的行业条件下,是进入还是退出行业,是增加还是减少投入,先进入某一有利可图的行业的厂商比后进入的厂商可赚取更多的利润,先退出某无利可赚的行业的厂商可节省很多资金。因此,最终经济利润为零的长期均衡并不会影响厂商进入行业。

(2)在长期中,所有的生产要素投入量都是可变的,厂商通过对全部生产要素投入量的调整来实现利润最大化的均衡条件MR?LMC。所以,零经济利润并不意味着厂商经营不好,而是表明厂商所在的行业是竞争性的。获得零经济利润的厂商没有必要退出该行业,因为零经济利润表明厂商的投资获得了正常利润。当然,投资者都希望获得正的经济利润,正是这一点激励着企业家的创新精神。

6.在20世纪初,美国有许多小的汽车生产厂家,20世纪末仅有三家大的厂家。假设这种情况不是反垄断法实施不力的结果,你怎样解释汽车制造厂家的数量减少?(提示:汽车行业的固有成本结构是怎样的?)

答:汽车制造厂家的数量减少与该行业的性质和成本结构有关,具体分析如下:

(1)自然垄断指某些行业或部门为了有效生产而只需要少数几个生产者或厂商的市场状况。这种行业可能始终呈现规模报酬递增的特征,若由多家厂商生产将产生较高的平均成本、造成社会资源的浪费。自然垄断的形成,使得少数几个大规模厂商能够依靠自己的规模经济来降低生产成本,使得规模经济的益处由该厂商充分加以利用。

(2)汽车产业是资本密集型产业,总成本中固定成本所占的比例比较大。汽车的单位成本随生产规模的增大而下降,具有规模经济。规模比较大的厂商能够以一个比较低的价格销售其产品。在长期中,一些规模比较小的厂商必然会面临无利可图的市场价格,从而退出985/211历年真题解析,答案,核心考点讲义,你想要的都在这→ 经济学历年考研真题及详解

Born to win

经济学考研交流群 <<<点击加入

行业。因此,在长期中汽车制造厂商数量会下降。在一定的产量水平下,规模经济效应会趋于消失,从而最终结果是行业中不只存在一家厂商。所以,自然的选择是较少的企业(比如题中三家大的厂家)以较低的平均成本,生产较多的产量,享受规模经济。实际上,长期以来汽车行业由于固定的成本结构形成自然垄断。

7.某行业是完全竞争的,该行业的每个厂商的经济利润为零。如果产品价格下降,没有厂商可以生存。你同意还是不同意这种认识?讨论一下。

答:不同意这种观点。原因如下:

(1)如果产品价格下降,所有厂商将会遭受经济损失。短期内,如果价格下降到总成本之下,可变成本以上,那么部分厂商将会降低产量并继续生产。因为短期内厂商的收益取决于短期利润,即短期收益与短期成本之差。而行业中厂商的短期成本是不同的。因此,只要价格不低于所有厂商的短期平均可变成本,从而短期内就总有厂商不会退出行业;只要价格不低于所有厂商的短期平均成本,就总有厂商可以获得正的经济利润,从而不会退出行业。

(2)在长期均衡点,完全竞争市场中每个厂商只能获得正常利润,得不到超额利润。当产品的价格下降时(由需求缩小引起的),长期均衡将被打破,会引起部分厂商退出该行业,由于部分厂商的退出,使该行业的供给量减少,供给曲线向左上方移动,从而使价格会上升到原来的水平,恢复到原来的价格水平时,留存下来的厂商又达到新的均衡。

因而不能说当价格下降时,没有厂商可以生存。

8.对音像电影的需求增加也可以增加演员的薪水。解释电影的长期供给曲线是水平的还是向上倾斜的。

答:电影的长期供给曲线是向上倾斜的。 长期供给曲线的形状取决于行业的成本结构,即需求量增加与减少对投入要素价格的影响程度,投入要素价格是厂商为生产过程而必须进行的支付。

如果演员的数量相对不变,随着音像电影需求的增加,为了邀请演员拍摄电影,必须支付更多的薪水,这使得音像电影行业的成本增加。因此,音像电影行业是成本递增的行业。在一个成本递增的行业,长期供给曲线是向上倾斜的。

9.电影的产出要总能使长期平均成本最小化。对还是错?试解释。 答:错误。其原因是:没有区分厂商生产的长期与短期决策。

在长期,在完全竞争的条件下,厂商的生产决策将使得长期平均成本最小。在长期,厂商会调整其资本和劳动投入组合,因而平均成本总是最小的。此外,自由进入与退出使得价格调整到等于最低平均成本的水平。

在短期,厂商的产量不一定是最优长期产量。因此,如果一些生产要素是固定的,厂商并不总是在长期平均成本最低的那一点进行生产。如图8-6所示。短期最优生产水平大于长期最优生产水平,所对应的产量并不是使长期成本最小化的产量。

985/211历年真题解析,答案,核心考点讲义,你想要的都在这→ 经济学历年考研真题及详解

Born to win

经济学考研交流群 <<<点击加入

图8-6

10.在具有向上倾斜的供给曲线的行业,能存在规模报酬不变吗?

答:在具有向上倾斜的供给曲线的行业,存在规模报酬不变。分析如下: 规模报酬不变是指一定比例的投入要素的增加导致产量同比例的增加。但是,当所有的企业增加投入要素的使用量时,一些投入要素的价格会上升,因为它们具有一条向右上方倾斜的供给曲线。比如,生产需要用到一些稀缺的资源,当产量增加时,投入要素的成本会上升。此时,双倍的投入依然可以有双倍的产出水平,但是由于要素价格上升,生产成本的上升将超过两倍。在这种情况下,该行业就是一个成本递增的行业,它具有向上倾斜的供给曲线。因此,一个行业可以同时具有规模报酬不变和一条向上倾斜的供给曲线。

11.市场是完全竞争的必要假设有哪些?根据本章所学,解释为什么这些假设都是重要的。

答:完全竞争又称为纯粹竞争,指不存在任何阻碍和干扰因素的市场情况,亦即没有任何垄断因素的市场结构。

(1)完全竞争市场的假设 ①价格接受者。市场上有大量的买者和卖者,任何单个个人和厂商都不能对市场施加看得见的影响,都是市场价格(市场的供给和需求决定)的被动接受者。

②产品同质。所有的生产者提供的都是同质的无差异的产品。 ③自由进入与退出。厂商进入和退出市场不会有特别的成本产生。 (2)这些假设都非常重要的原因

①价格接受者和产品同质假设保证了单一市场价格的存在,使市场供求分析变得有意义;决定了厂商的需求曲线是等于价格的一条水平线,从而决定了短期均衡条件MC?P。因为每一产品的市场价格既定,而且产品同质,厂商不可能通过产品差别化来竞争,只可能以相同的市场价格提供同质的商品,所以厂商的边际收益等于市场价格,即MR?P,所以追求利润最大化的MC?MR条件变成了MC?P。

②自由进入和退出假设影响行业的供给与价格,决定了完全竞争条件下的长期均衡条件P?LMC,使长期中厂商的经济利润为零。

③这些假设保证了每一个竞争性的市场均衡是帕累托最优。竞争性市场中的激励机制同样迫使生产者高效率地组织生产,否则“优胜劣汰”的法则使其不能生存下去。同样,这些激励机制将鼓励企业的技术创新,并将这些创新最佳地使用于企业生产活动中,为企业带来更大的收益。

985/211历年真题解析,答案,核心考点讲义,你想要的都在这→ 经济学历年考研真题及详解

Born to win

经济学考研交流群 <<<点击加入

12.假设一个竞争性行业面临着需求的增长(即曲线向上移动)。一个竞争市场保证产出增加的步骤是什么?如果政府制定了最高限价,你的答案会不会改变?

答:(1)假设需求增长,竞争性市场保证产出增加的步骤是:

①需求增长,需求曲线向上移动和短期供给曲线相交形成短期均衡,均衡价格上升。 ②价格上升使每个厂商增加产量,这时每个厂商获得正的经济利润。 ③随着时间的推移,正的经济利润吸引新厂商进入(假设新厂商与原有厂商的成本结构相同),供给量增加,使供给曲线向右移动,直至在一个新的价格水平下达到长期均衡。每个厂商在有效规模处生产,行业供给量增加。

(2)最高限价的情况有所不同

最高限价是指政府为了防止某些产品的价格上涨而规定的这些产品的最高价格,且最高价格总是低于市场均衡价格。

当需求增长时,因为政府已经采取了最高限价,价格不会发生改变,那么厂商就不会增加产出,从而经济利润不会增加,不会有新厂商进入,从而供给曲线也不发生移动。因此,需求增长在最高限价的情况下,不会使产出增加,而是代之以产品的短缺。

13.政府通过了一项法令,允许对每亩用于种植烟草的土地给予补贴。这一措施会怎样影响烟草的长期供给曲线?

答:补贴将会使长期供给曲线向右移动。

政府给予补贴相当于等量地降低了企业的平均成本和边际成本,每个企业增加产量,存在正的经济利润,诱使新企业进入,供给量增加,供给曲线向右移动,价格下降,直至价格达到新的长期均衡,在新的长期均衡条件下,减掉补贴以后的边际成本等于新的均衡价格,每个企业在有效规模处生产。

14.一个地区有若干家商店销售某种品牌的真空吸尘器。

(1)假如所有销售商要价相同,他们的长期经济利润是否都等于零?

(2)假如所有销售商要价相同,但是一家当地销售商的商店经营场所为他自己拥有的房子,不必付租金,这个销售商的经济利润为正吗?

(3)不必付租金的这家销售商是否有激励去降低其销售价格? 答:(1)如果所有销售商要价相同,它们的长期经济利润都等于零。因为,如果经济利润大于零,则行业外的其他厂商会进入该行业;如果经济利润小于零,则行业中的某些厂商会退出该行业。

(2)这个销售上的经济利润仍然为零。如果这个销售商不支付租金,他使用经营场所的会计成本为零,但是仍然存在机会成本,该机会成本将使得经济利润为零。

(3)该销售商没有激励去降价,因为在当前的价格水平下,销售商能够卖出他想卖的任意数量,降价会降低他的经济利润。如果所有厂商销售同质的商品,则他们将索要相同的价格。因而降价后,该销售商将不再是利润最大化。

8.3 课后练习题详解

1.下表给出了一家厂商的单位产品销售价格(单位:美元)和总成本的信息。 (1)填充表中空格。

985/211历年真题解析,答案,核心考点讲义,你想要的都在这→ 经济学历年考研真题及详解

Born to win

经济学考研交流群 <<<点击加入

(2)如果价格从60美元下降到50美元,企业的产量选择和利润将如何变化? 答:(1)如表8-1所示。

表8-1

(2)表8-1给出了当价格从60美元下跌至50美元时,企业的收益与成本信息。 当价格为50美元,生产单位为9时,企业获得最大利润;当价格为60美元,生产单位为10时,企业获得最大利润。所以,当价格从60美元下降到50美元,企业的产出从10单位下降至9单位,利润从190美元跌至95美元。

2.从表中的数据分析,如果生产固定成本从100美元增加到150美元,再增加到200美元,给定价格每单位60美元保持不变,厂商的产出选择和利润会发生什么变化?你从固定成本对厂商的产出选择的影响能得出什么一般结论?

答:表8-2给出了当固定成本分别为100美元、150美元、200美元时,厂商的收益与成本信息。

表8-2 985/211历年真题解析,答案,核心考点讲义,你想要的都在这→ 经济学历年考研真题及详解

Born to win

经济学考研交流群 <<<点击加入

当固定成本分别为100美元、150美元、200美元时,厂商为追求利润最大化(损失最小化)的产出选择都是生产10个单位。

从该题的结论可以看出,固定成本对厂商的产出选择没有影响。厂商追求利润最大化的条件是MC?MR,但固定成本FC并不影响MC。固定成本越高导致利润越低,但是所有的利润最大化结果都出现在同样的产出水平上,即10单位产出。

3.使用与练习1相同的信息。 (1)推导厂商的短期供给曲线。(提示:画出成本曲线的基本形状。) (2)假如市场中有100家相同的厂商,市场供给曲线是怎样的? 答:(1)在完全竞争市场上,厂商的短期供给曲线就是平均可变成本AVC曲线最低点以上的边际成本MC曲线。

表8-3列了厂商的边际成本MC、总成本TC、可变成本VC、固定成本FC,以及平均可变成本AVC的信息。

表8-3

根据表8-3中数据描点画图,作出MC曲线、AVC曲线如图8-7(1)所示。 厂商的短期供给曲线即MC与AVC交点右边的MC曲线(8单位以右的部分)。

985/211历年真题解析,答案,核心考点讲义,你想要的都在这→ 经济学历年考研真题及详解

Born to win

经济学考研交流群 <<<点击加入

图8-7(1) 边际成本与平均可变成本曲线

(2)在完全竞争市场上,行业的短期供给曲线是私人厂商的供给曲线在水平方向上的加总,即生产这种产品的所有企业短期供给曲线的水平坐标之和。市场上有100家相同的厂商,则应将私人厂商的短期供给曲线的纵坐标不变,横坐标变为原来的100倍,如图8-7(2)所示。

图8-7(2) 完全竞争市场的行业短期供给曲线

4.假设你是竞争市场上一家钟表制造厂的经理,你的生产成本为:C?200?2q2,式中,q为产出水平;C为总成本(生产的边际成本为4q,固定成本为200美元)。

(1)如果价格是100美元,为追求利润最大化,你应该生产多少钟表?

(2)利润是多少?

(3)价格最低为多少时,厂商可保持正的产出? 解:(1)制造厂的利润函数为:

??pq?C?100q?200?2q2

利润最大化的一阶条件为:

d??100?4q?0 dq解得:q?25。

(2)将q?25代入利润函数,可得:

??100?25??200?2?252??1050(美元)

985/211历年真题解析,答案,核心考点讲义,你想要的都在这→ 经济学历年考研真题及详解

Born to win

经济学考研交流群 <<<点击加入

(3)在短期,只要收益TR比总可变成本TVC大,厂商就会生产正的产出。而厂商的短期供给曲线就是平均可变成本TVC曲线最低点以上的边际成本MC曲线。

VC2q2AVC???2q

qqMC?4q

因此,只要q?0,就有MC?AVC,即:只要价格为正,厂商在短期内就会生产正的产出。

5.假设竞争性厂商生产的产量为q时的边际成本由下式给出:MC?q??3?2q,如果该产品的市场价格为9美元。回答下列问题:

(1)厂商的产出水平是多少? (2)厂商的生产者剩余是多少?

(3)假设厂商的平均可变成本曲线为:AVC?q??3?q,已知厂商的固定成本为3美元,短期内厂商的利润为正、为负还是为零? 解:(1)在完全竞争市场上,利润最大化的条件为:P?MC,即:9?3?2q,解得:q?3,即厂商的产出水平为3。

(2)生产者剩余是市场价格以下边际成本曲线以上的区域面积。

因为MC是线性变化的,所以生产者剩余(PS)是如图8-8所示的阴影三角形的面积。

PS?0.5?6?3?9(美元)

图8-8 生产者剩余

(3)由题意可知,总成本函数为:

TC?q???3?q?q?3

利润为:

??TR?TC?6q?q2?3?6(美元)

故短期内厂商的利润为正。

6.竞争性市场中某厂商的生产成本函数为C?50?4q?2q2,边际成本函数为MC?4?4q。当市场价格给定为20美元时,厂商产量为5,厂商利润达到最大化了吗?在

985/211历年真题解析,答案,核心考点讲义,你想要的都在这→ 经济学历年考研真题及详解

Born to win

经济学考研交流群 <<<点击加入

长期,厂商产量应该定为多少?

解:厂商的利润函数为:

??pq?C?20q??50?4q?2q2?

利润最大化的条件为:

d??16?4q?0 dqq?4

厂商未达到利润最大化,在长期厂商达到利润最大化或亏损最小化的产量为4。 此时的利润为:

?? 20?4??50?4?4?2?42???18

因此,在产品价格或厂商的成本结构没有变化情况下,由于在价格等于边际成本的产量水平上(利润最大化时),经济利润是负的,所以厂商在长期内应该退出该行业。

7.假设厂商成本为C?q??16?4q2。

(1)求出平均成本、固定成本、可变成本、平均可变成本和平均固定成本。(提示:边际成本为MC?8q。)

(2)在图上画出平均成本、边际成本和平均可变成本曲线。

(3)求出最小化平均成本的产出水平。 (4)当价格在哪一区间时厂商产量为正? (5)当价格在哪一区间时厂商利润为负? (6)当价格在哪一区间时厂商利润为正? 答:(1)从厂商的成本函数可知,该厂商的可变成本和固定成本分别为:

VC?4q2,FC?16

因而该厂商的平均成本、平均可变成本和平均固定成本分别为:

AC?C?q?qVCAVC??4qqFC16AFC??qq?4q?16q

(2)平均成本、边际成本和平均可变成本曲线如图8-9所示。

985/211历年真题解析,答案,核心考点讲义,你想要的都在这→ 经济学历年考研真题及详解

Born to win

经济学考研交流群 <<<点击加入

图8-9 成本曲线

(3)当AC?MC时,平均成本最小,即有:

16AC?4q??8q?MC

q解得:q?2。

(4)当P?MC?AVC时,厂商就会提供正的产量。如图8-9所示,MC曲线始终位于AVC曲线的上方,因此,厂商在任何正的价格处都将提供正的产量。

(5)当P?MC?AC时,厂商所获得的利润将为负。当MC?AC时,由(3)可知,q?2,AC?16。因此,当价格低于16时,厂商的利润将为负。

(6)由(5)可知,当价格低于16时,厂商的利润为负。因此只要价格高于16,厂商就可以获得正的利润。

8.一家竞争性厂商的短期成本函数如下:

C?q??q3?8q2?30q?5

(1)求出MC、AC和AVC,并在图形上表示出来。 (2)当价格位于哪一区间时厂商产量为零? (3)在图上标出厂商的供给曲线。 (4)当价格为多少时企业产量为6? 答:(1)厂商的边际成本、平均成本和平均可变成本分别为:

MC?dC?3q2?16q?30dqC5AC??q2?8q?30?

qqVCAVC??q2?8q?30q相应的成本曲线如图8-10所示。

985/211历年真题解析,答案,核心考点讲义,你想要的都在这→ 经济学历年考研真题及详解

Born to win

经济学考研交流群 <<<点击加入

图8-10 成本曲线及其关系

(2)在短期内,只要价格不低于最低平均可变成本,厂商进行生产就是有利可图的。如果价格低于最低平均可变成本,则厂商在短期内停止营业,因为此时它仅损失固定成本,而不是固定成本加某些可变成本。当MC?AVC时,平均可变成本最小。因而有:

AVC?q2?8q?30?3q2?16q?30?MC

解得:q?4。

因而最小平均可变成本为:AVC?42?8?4?30?14。 因此,当价格P?14时,厂商产量为零。

(3)厂商的供给曲线为位于AVC最低点上侧的MC曲线,如图8-10中的A点右端的MC曲线。

(4)由P?MC可得:

P?MC?3q2?16q?30?3?62?16?6?30?42

因此,当价格为42时,厂商产量为6。

9.(1)假设某厂商的产量函数为q?9x1/2,在短期,固定成本为1000美元,x为可变投入,其成本为4000美元/单位。生产q单位产品的总成本为多少?[即求出总成本函数

C?q?]

(2)写出供给曲线方程。

(3)如果价格为1000美元,厂商产量为多少?利润水平为多少?在成本曲线图上表示出你的结论。

解:(1)厂商的总成本函数为:C?x??FC?VC?1000?4000x。 q2因为产量函数为:q?9x,所以有:x?。

811/2代入成本函数可得:

4000q2C?q???1000

81(2)厂商供给曲线是MC曲线中MC?AVC的部分。MC?整个边际成本曲线即为供给曲线:

8000q4000q,所以?AVC?8181985/211历年真题解析,答案,核心考点讲义,你想要的都在这→ 经济学历年考研真题及详解

Born to win

经济学考研交流群 <<<点击加入

p?8000q?q?0? 81(3)如果价格为1000美元,则有:

p?8000q?1000?q?10.125 81此时,厂商的利润为:

??1000?10.125?1000?4000?10.1252?81?4062.5

厂商在价格P?1000美元下的利润如图8-11中的阴影部分面积所示。

图8-11 利润的图示

10.假设某一特殊行业的信息如下:

QD?6500?100P 市场需求 QS?1200P 市场供给 C?q??722?q2/200 厂商总成本 MC?q??2q/200 厂商边际成本

假定所有厂商完全同质,而且市场是完全竞争的。

(1)计算均衡价格、均衡产量、厂商供给产量和每家厂商的利润。

(2)在长期将有厂商进入还是退出市场?试解释。进入或退出将会对市场产生何种影响?

(3)在长期,每家企业销售其产品的最低价格为多少?利润是正的、负的还是为零?解释你的结论。

(4)在短期,每家企业销售其产品的最低价格为多少?利润是正的、负的还是为零?解释你的结论。

解:(1)由市场供给与需求相等可得:

QD?6500?100P?QS?1200P

因而可以解得市场均衡价格为:P?5;均衡产量为QS?1200P?6000。

985/211历年真题解析,答案,核心考点讲义,你想要的都在这→ 经济学历年考研真题及详解

Born to win

经济学考研交流群 <<<点击加入

厂商供给的产量满足P?MC,即:5?2q,因而厂商的供给产量为:q?500。 2005002每家厂商的利润为:??Pq?C?q??5?500?722??528。

200此时,市场中厂商的数目为:n?6000?500?12(家)。

(2)在长期中,将有企业进入市场,因为行业中现有企业的利润为正。随着新厂商的进入,供给曲线将向右移动,因而均衡价格将下降,这将使厂商的利润减为零,从而没有新企业有进入的激励。

(3)在长期,厂商不会以低于最低平均成本的价格销售其产品。在任何低于最低平均成本的价格下,厂商的利润为负,此时厂商最好出售其固定投入退出该市场,产量减至零。

当MC?AC时,厂商的平均成本最低,即有:

2q722q ??200q200解得:q?380,因而最低平均成本为:AC?q?380??3.8。

因此,在长期,每家企业销售其产品的最低价格为3.8,此时其利润为零。

(4)在短期,在任何正的价格处,厂商都将销售其产品,因为在任何正的价格处,边际成本(MC?q/100)都高于平均可变成本(AVC?q/200)。只要价格低于最小平均成本3.8,厂商的利润就是负的。

11.竞争性厂商的总成本函数为C?q??450?15q?2q2,边际成本函数为

MC?q??15?4q。如果市场价格为P?115美元/单位,求出厂商生产的产量,并计算利润和

生产者剩余。

解:竞争性厂商的利润函数为:

??pq?C?115q??450?15q?2q2?

利润最大化的条件为:

d??100?4q?0 dq解得企业的产量为:q?25。

此时利润为:??115?25?450?15?25?2?252?800(美元)。 生产者剩余等于利润加上固定成本,即800+450=1250美元。

12.很多商店都提供相片冲洗服务。如果每家商店冲洗胶卷的成本函数均为

C?q??50?0.5q?0.08q2,边际成本为MC?q??0.5?016.q,则:

(1)如果冲洗一卷胶卷收费8.50美元,该行业处在长期均衡状态吗?

(2)假设现在有一种新的冲洗技术出现,该技术可以使得冲洗相片的成本降低25%。如果行业目前正处于长期均衡状态,则厂商愿意支付多高的价格来购买这种新技术?

解:(1)由P?MC可得:

MC?0.5?0.16q?8.5?P 985/211历年真题解析,答案,核心考点讲义,你想要的都在这→ 经济学历年考研真题及详解

解得:q?50。

Born to win

经济学考研交流群 <<<点击加入

此时每家商店的利润为:??8.5?50??50?0.5?50?0.08?502??150(美元)。 因而可以断定该行业没有处于长期均衡,因为利润大于零。

在长期均衡时,厂商将在价格等于最低平均成本处进行生产,因而利润为零,没有厂商有激励进入或退出市场。

50由MC?0.5?0.16q??0.5?0.08q?AC可得:q?25。

q长期均衡的价格为:P?4.5(美元)。

(2)如果采用此新技术,则厂商的成本变为:

Cnew?q??0.75??50?0.5q?0.08q2??37.5?0.375q?0.06q2

MCnew?q??0.375?0.12q

由长期价格P?4.5?MCnew?q?可以解得,q?34,此时厂商的利润为33.39美元。 因此,厂商最多愿意支付33.39美元用于购买这种新技术。

13.假设某城市市中心有很多家热狗销售亭,每家销售亭的边际成本均为1.50美元/个,没有固定成本,每家销售亭每天最多能销售100个热狗。

(1)如果热狗价格为2美元,每家销售亭每天想销售多少个热狗?

(2)如果该行业是完全竞争的,2美元的价格是否会一直持续下去?如果不会,则价格为多少时会达到均衡?

(3)如果每家销售店每天都销售100个热狗,市场需求函数为Q?4400?1200P,市场中有多少家销售亭?

(4)如果市政府决定通过无偿发放许可证的方式来对热狗销售店进行管理,如果一共发放了20个许可证,每家销售亭的日最高销售量仍然为100个,热狗的销售价格为多少?

(5)如果市政府决定出售许可证,每家销售亭愿意支付的最高价格为多少? 解:(1)因为边际成本为1.5美元/个,而价格为2美元/个,每家销售亭会尽可能的多销售,即销售100个热狗。

(2)如果该行业是完全竞争的,则2美元的价格不会永远持续下去。因为此时每个销售亭都有激励将价格略低于2美元,从而抢占更多的消费者,销售更多的热狗。此外,新的热狗销售亭也会开张。价格最终将下降为1.5美元,与边际成本相等。

(3)市场均衡时,价格为P?MC?1.5,市场销量为:Q?4400?1200?1.5?2600。

如果每家销售店销量为100,则销售店的数目为:2600÷100=26(家)。 (4)此时热狗的市场供给量为:20×100=2000。 此时热狗售价为:P??4400?2000??1200?2(美元)。

(5)由于在许可证下,热狗的售价为2美元,因此每个热狗获利0.5美元。因此,每家销售亭愿意支付的最高价格为:0.5×100=50(美元)。

14.对竞争性行业中的一个厂商的售价为5美元的产品征收销售税为1美元。 (1)税收是如何影响厂商的成本曲线的? 985/211历年真题解析,答案,核心考点讲义,你想要的都在这→ 经济学历年考研真题及详解

Born to win

经济学考研交流群 <<<点击加入

(2)厂商的价格、产出、利润会发生哪些变化? (3)是否会有厂商进入或退出该行业? 答:(1)由于对单个企业征收1美元的税,它的全部成本曲线上移一个单位。总成本变动为TC?q,边际成本变动为MC?1,平均成本变动为AC?1。

(2)在完全竞争市场上,厂商是价格接受者,因此只对单个厂商征税并不改变市场价格。厂商是价格接受者,因此厂商的价格保持不变。

由于单个厂商的短期供给曲线是高于平均可变成本的边际成本曲线,而边际成本曲线上移了,即短期供给曲线上移。因此,对于每个价格,该厂商供给的产品减少了,即产出降低。

厂商的利润为???P?MC?Q,价格P不变,而边际成本MC增大,产出Q减小,故利润减小。

(3)长期内,完全竞争市场上,厂商的经济利润为零(P?MC),如果对单个厂商征税,使其MC?P,必将导致利润为负,厂商亏损,因此该厂商将被迫关闭,退出该行业。

15.在一竞争性行业,有一半的厂商(环境污染者)被征收10%的销售税,这些税收收入用于对其他厂商(没造成污染者)的销售额补贴10%。请回答:

(1)在销售税补贴政策之前,假设所有厂商有相同的不变的长期平均成本,你认为在短期和长期,产品价格、每个厂商的产出和整个行业的产出各会发生什么变化?(提示:价格与行业投入要素间的关系是怎样的?)

(2)这种税收收入等于补贴支出的预算平衡政策是否总能达到?解释为什么。 答:(1)商品价格依赖于行业中所有厂商生产的数量。销售税征收和补贴的实行,在短期会造成污染厂商生产数量下降,非污染厂商生产数量上升。因为行业中污染厂商和非污染厂商的数量相等,并且所有厂商有相同的不变长期平均成本,各个厂商的短期供给曲线的形状是相同的,所以,实行政策后,污染者的产量降低将与非污染者的产量增加相等,行业产出不变,价格保持不变。

如果存在长期均衡,则在实行政策以前,价格=边际成本=最低长期平均成本,实行政策后,污染者的价格<长期平均成本,长期内,他们会退出行业;非污染者的价格>长期平均成本,长期内,会有新的非污染厂商进入行业。最终达到新的长期均衡,行业中将没有污染者。非污染者享受补贴,相当于边际成本下降,达到长期均衡时,价格将会等于新的边际成本,因此价格下降,行业的产出和单个厂商的产出都有提高。

(2)这种税收收入等于补贴支出的预算平衡政策不是总能达到的。

在长期,非污染者进入,污染者退出,从污染者获得的收入减少,为非污染者提供的补贴增加,由于随着污染者退出和非污染者的进入,税收越来越少而补贴逐渐增加,税收收入将会小于补贴支出。从(1)的分析中可以看出,长期内,从第一个污染者退出,或者第一个新的非污染者进入时起,这种预算平衡就被打破了。

如果想要维持预算平衡,那么税收和补贴就必须随着厂商的进入和退出做出调整。由于从污染者处获取的税收不断缩减,因此非污染者获得的补贴将不断降低,最终,税收和补贴都将为零。

以上内容为跨考网整理的经济学考研课后习题答案解析的一部分,限于篇幅原因,如果同学还想获得更多经济学课后习题资料,可以关注微信公众平台索要经济学考研资料,你想

985/211历年真题解析,答案,核心考点讲义,你想要的都在这→ 经济学历年考研真题及详解

Born to win

经济学考研交流群 <<<点击加入

要的资料都在这儿→jjxkyzs。 想了解经济学考研高分的秘密吗?请点击>>>:经济学考研解题技巧

跨考经济学考研辅导提醒您: 成功的原因千千万,失败的原因就那么几个,加入我们的经济学考研交流群,考研经验交流,规避风险,锁定名校一次进!

985/211历年真题解析,答案,核心考点讲义,你想要的都在这→ 经济学历年考研真题及详解

本文来源:https://www.bwwdw.com/article/kmmp.html

Top